Men's and women's health ACCP Questions & Answers Flashcards

1
Q
  1. M.T. is a 72-year-old white man (height 69 inches,
    weight 68 kg) with a history of hypertension who
    admits smoking one pack of cigarettes per day
    and having a poor diet. He states that he walks on
    his treadmill 30 minutes a day. His bone mineral
    density (BMD) T-score is –2.1 at the hip and –2.2
    at the spine. His fracture history includes a fall at
    age 68 with an ankle fracture. His Fracture Risk
    Assessment Tool (FRAX) score (10-year major
    fracture probability) is 14%, and his probability of
    hip fracture is 6.7%. Which best describes M.T.’s
    condition?
    A. He has low bone mass (osteopenia) of the hip
    and spine.
    B. He has osteoporosis of the spine and low bone
    mass (osteopenia) of the hip.
    C. He has osteoporosis of the hip and spine.
    D. He has normal BMD of the hip and spine.
A
  1. Answer: A
    The patient’s T-score at the hip and spine are between -1
    and -2.5 SD, which is considered low bone mass (Answer
    A is correct). Normal BMD is defined as a T-score
    within 1 SD of the young adult mean, which the patient
    does not have (Answer D is incorrect). Osteoporosis is
    defined as having BMD with a T-score of at least -2.5
    SD, which the patient does not have in either the hip or
    the spine (Answers B and C are incorrect).
How well did you know this?
1
Not at all
2
3
4
5
Perfectly
2
Q
  1. Which treatment is best for M.T.?
    A. Take calcium 1200 mg orally daily, vitamin D
    800 international units orally daily, and alendronate
    70 mg orally weekly.
    B. Take calcium 1200 mg orally daily and vitamin
    D 600 international units orally daily, and
    begin weight-bearing exercise.
    C. Take calcium 1200 mg orally daily, vitamin D
    600 international units orally daily, and raloxifene
    60 mg orally daily.
    D. Take calcium 1200 mg orally daily, vitamin
    D 400 international units orally daily, and
    risedronate 35 mg orally weekly, and begin
    weight-bearing exercise.
A
  1. Answer: A
    Even though the patient is currently thought to have
    osteopenia, his 10-year probability of a major osteoporosis-
    related fracture is 14%, and his 10-year
    probability of a hip fracture is 6.7% according to the
    FRAX score, which indicates a need for drug therapy
    (Answer A is correct). Bisphosphonates such as
    alendronate and risendronate are considered first-line
    drugs because they inhibit normal and abnormal bone
    resorption and reduce vertebral and non-vertebral fractures
    by 30%–50%. Raloxifene is indicated for preventing
    osteoporosis in postmenopausal women. It works
    as a selective estrogen receptor modulator and is best
    for preventing vertebral fractures, not hip fractures
    (Answer C is incorrect). Calcium and vitamin D are
    both recommended for this patient as well. Because of
    the patient’s age and poor diet, the recommended dosage
    of vitamin D is 800 international units orally per
    day (Answer A is correct; Answer D is incorrect); 600
    international units orally per day is recommended for
    those younger than 70 years (Answer B is incorrect).
    Answer B is also incorrect because the patient requires
    pharmacotherapy and not just calcium and vitamin D.
    He walks 30 minutes/day, which satisfies the recommendation
    of weight-bearing activity (Answers B and
    D are incorrect).
    Drug therapy begins with the following:
    (a) A hip or vertebral fracture
    (b) BMD T-score below -2.5 at the femoral neck or
    spine, excluding secondary causes
    (c) BMD T-score between -1.0 and -2.5 at the femoral
    neck or spine and a 10-year probability of hip
    fracture 3% or greater or 10-year probability of
    major osteoporosis-related fracture 20% or greater
    according to the FRAX system.
How well did you know this?
1
Not at all
2
3
4
5
Perfectly
3
Q
  1. A 29-year-old woman (height 65 inches, weight
    63 kg) has a history of two deep venous thromboses
    (DVTs) but is otherwise healthy; she is seeking
    to become pregnant. She currently takes warfarin 3 mg orally daily. Which regimen is the best recommendation
    for this patient?
    A. Continue current warfarin dose to prevent
    clots during pregnancy.
    B. Continue warfarin therapy but increase the
    dose to prevent clots during pregnancy.
    C. Discontinue warfarin; start enoxaparin 60 mg
    subcutaneously every 12 hours until pregnant
    and continue through pregnancy.
    D. Discontinue warfarin; start heparin 5000 units
    subcutaneously every 8 hours daily until 12
    weeks pregnant, and then reinitiate warfarin.
A
  1. Answer: C
    Warfarin is contraindicated in pregnancy and a known
    teratogen (Answers A, B, and D are incorrect). Warfarin
    should not be used at any time during pregnancy unless
    the benefit outweighs the risk in very rare and special
    cases. If a woman needs anticoagulation and is planning
    to conceive or is pregnant, she should find an alternative
    anticoagulant such as a low-molecular-weight
    heparin (e.g., enoxaparin) or heparin (although with
    heparin, there may be a risk of osteoporosis with
    extended duration of use (Answer D is incorrect). A
    low-molecular-weight heparin would be the agent of
    choice (Answer C is correct).
How well did you know this?
1
Not at all
2
3
4
5
Perfectly
4
Q
  1. J.K. is a 51-year-old postmenopausal woman with
    severe hot flashes that have not resolved with
    venlafaxine 75 mg orally daily. She is otherwise
    healthy, with no history of cancer and no surgical
    procedures. She is given conjugated estrogen
    0.625 mg orally daily. Which treatment is best for
    J.K.?
    A. No other drug is needed; estrogen alone is sufficient
    for hot flashes.
    B. No other drug is needed; estrogen should be
    discontinued, and she should continue on
    venlafaxine.
    C. Venlafaxine should be continued and
    Medroxyprogesterone acetate should be added
    to decrease the risk of stroke.
    D. Venlafaxine should be discontinued and
    Medroxyprogesterone acetate should be added
    to decrease the risk of endometrial cancer.
A
  1. Answer: D
    Medroxyprogesterone acetate is added to conjugated
    estrogens to decrease the risk of endometrial cancer;
    venlafaxine is not relieving her hot flashes, therefore it
    should be discontinued (Answer D is correct). The addition
    of medroxyprogesterone acetate does not decrease
    the risk of stroke (Answer C is incorrect). Estrogen
    alone is insufficient because the patient has an intact
    uterus, as indicated by her medical history (no surgical
    procedures) (Answer A is incorrect). Venlafaxine is not
    relieving her hot flashes; therefore, it should be discontinued
    (Answer B is incorrect).
How well did you know this?
1
Not at all
2
3
4
5
Perfectly
5
Q
  1. C.S. is a 49-year-old postmenopausal woman experiencing
    severe hot flashes, vaginal dryness, and
    pain during sexual intercourse. C.S. has a history of
    irregular heavy uterine bleeding, which resulted in
    a total hysterectomy 5 months ago. Her hot flashes
    are affecting her quality of life. Which treatment is
    best to recommend for C.S.?
    A. Estradiol vaginal cream 0.1 mg/g.
    B. Conjugated estrogen and medroxyprogesterone
    acetate 0.625 mg/2.5 mg tablets.
    C. Conjugated estrogen 0.3-mg tablets.
    D. Ospemifene 60-mg tablets.
A
  1. Answer: C
    The patient is experiencing systemic symptoms such as
    hot flashes, and she has localized genitourinary atrophy,
    which probably results in pain during sexual intercourse.
    Estradiol vaginal cream and ospemifene are
    indicated for genitourinary atrophy, not for vasomotor
    symptoms (Answers A and D are incorrect). The best
    treatment would be an oral or transdermal systemic
    agent (Answer C is correct). The patient has had a hysterectomy;
    therefore, a progestogen in combination
    with estrogen is unnecessary (Answer B is incorrect).
How well did you know this?
1
Not at all
2
3
4
5
Perfectly
6
Q
  1. S.F. is a 20-year-old woman initiated on ethinyl
    estradiol 30 mcg/drospirenone 3 mg oral tablets
    5 months ago for contraception. She was recently
    prescribed lamotrigine for bipolar disorder. Which
    best describes the drug interaction that may occur
    with ethinyl estradiol/drospirenone and lamotrigine
    in this patient?
    A. The effectiveness of ethinyl estradiol and
    drospirenone may be decreased.
    B. The effectiveness of lamotrigine may be
    increased.
    C. The effectiveness of lamotrigine may be
    decreased.
    D. The effectiveness of ethinyl estradiol and
    drospirenone may be increased.
A
  1. Answer: C
    The efficacy of lamotrigine may be decreased (Answer
    C is correct; Answer B is incorrect). Estrogen and drospirenone
    are not affected by lamotrigine (Answers A
    and D are incorrect). (See Answer 6 table.)
How well did you know this?
1
Not at all
2
3
4
5
Perfectly
7
Q
  1. A study compares the incidence of herpes simplex
    genital infections in patients receiving suppressive
    therapy with acyclovir versus valacyclovir. After
    1 year of follow-up, 25% in the acyclovir group and
    20% in the valacyclovir group experience a recurrent
    infection (p<0.05). Which best represents how
    many patients (in 1 year) would need to be treated
    with valacyclovir over acyclovir to prevent one
    recurrent infection?
    A. 5.
    B. 20.
    C. 25.
    D. There is insufficient information to calculate
    this number.
A
  1. Answer: B
    The number of patients needed to treat with valacyclovir
    over acyclovir to prevent one recurrent HSV genital
    infection (1 year of follow-up of study participants
    receiving suppressive therapy [acyclovir or valacyclovir],
    with 25% and 20%, respectively, experiencing a
    recurrent infection) is 20 = 1/(0.25−0.20) (Answer B
    is correct; all other answers are incorrect). The only
    information needed is the absolute risk in both groups
    (which is provided).
How well did you know this?
1
Not at all
2
3
4
5
Perfectly
8
Q
  1. K.M. is a 28-year-old woman (height 68 inches,
    weight 98 kg) with a history of migraine with aura
    who is seeking contraception. Her blood pressure
    today is 135/82 mm Hg; she denies smoking and
    alcohol use and states that she would like to have
    children in a year or so. Which is the best contraceptive
    agent for K.M.?
    A. Levonorgestrel intrauterine system.
    B. Oral norethindrone tablet.
    C. Transdermal ethinyl estradiol/etonogestrel
    patch.
    D. Oral ethinyl estradiol/desogestrel tablet.
A
  1. Answer: B
    The patient has a history of migraine with aura, which
    precludes any estrogen product (oral ethinyl estradiol/
    desogestrel oral tablet and transdermal contraceptive
    patch). Her blood pressure is slightly elevated but not
    greater than 160/100 mm Hg, which would not be a contraindication
    for estrogen use; however, her migraine
    with aura rules out the use of estrogen products
    (Answers C and D are incorrect). She also has obesity
    at 215 lb (98 kg) and weighs more than 90 kg, so the
    patch is not recommended (again, Answer C is incorrect).
    The levonorgestrel IUS (Mirena, Kyleena, Skyla,
    Liletta) may be an option, but the patient is interested
    in conceiving in a year or so, which makes the IUS not
    cost-effective because they last 3–6 years. (Answer A
    is incorrect). Norethindrone oral tablet is a progestogen-
    only pill and the best choice (Answer B is correct).
How well did you know this?
1
Not at all
2
3
4
5
Perfectly
9
Q
  1. L.L. is a 38-year-old woman who has been trying
    to conceive for the past 7 months. Her husband’s
    medical examination is normal; L.L. is not ovulating
    every month. She has not tried any medications
    previously to induce ovulation. Which medication
    is best to initiate in L.L. to induce ovulation?
    A. Human chorionic gonadotropin (hCG).
    B. Nafarelin/gonadotropin-releasing hormone
    (GnRH) agonist.
    C. Human menopausal gonadotropin (hMG).
    D. Clomiphene.
A
  1. Answer: D
    Clomiphene citrate is the first-line choice to stimulate
    ovulation (Answer D is correct). Another first-line
    option could be an aromatase inhibitor such as letrozole;
    however, this was not an answer choice. Menopur
    (hMG), Synarel (GnRH agonist), and Ovidrel (hCG)
    are not first-line agents and are usually used after clomiphene
    citrate or aromatase inhibitors have failed or
    when the patient is undergoing assisted reproductive
    therapies (Answers A, B, and C are incorrect).
How well did you know this?
1
Not at all
2
3
4
5
Perfectly
10
Q
  1. T.G. is a 22-year-old woman who comes to a community
    pharmacy and asks about emergency contraception
    (EC). She states that she was out of town
    for the weekend and was swimming when her contraceptive
    vaginal ring slipped out. She has been
    without the ring for 3 days because she did not have
    a new one with her for replacement. She states she
    had unprotected intercourse 4 nights ago. She is
    worried about becoming pregnant. Which is the
    best recommendation for T.G.?
    A. Recommend that she see her physician for a
    levonorgestrel 1.5 mg prescription.
    B. Recommend EC; it may still be effective
    because she is within the 120-hour time
    window.
    C. Do not recommend EC; it may be ineffective
    because she is beyond the 72-hour time window.
    D. Do not recommend EC; instead, recommend
    that she insert a new contraceptive vaginal ring.
A
  1. Answer: B
    Clinical studies suggest that levonorgestrel emergency
    contraception is still effective for up to 120 hours after
    unprotected intercourse though package labeling only
    states 72 hours, and the patient does not need a prescription
    (Answer B is correct; Answer A is incorrect).
    She stated that her vaginal ring slipped out of place.
    If the vaginal ring had been out of place longer than 3 hours and unprotected intercourse had occurred,
    EC should have been recommended (Answers C and
    D are incorrect). When inserting a new vaginal ring,
    she should also be instructed to use a BUM for at least
    7 days (again, Answer D is incorrect).
How well did you know this?
1
Not at all
2
3
4
5
Perfectly
11
Q
  1. K.S. is a 45-year-old man who has difficulty maintaining
    an erection during intercourse. His medical
    history includes diabetes mellitus and hyperlipidemia.
    His drugs include aspirin, metformin, and
    pravastatin. Blood pressure is 130/81 mm Hg,
    hemoglobin A1C 6.2, total cholesterol 195 mg/dL,
    low-density lipoprotein cholesterol (LDL) 106 mg/
    dL, high-density lipoprotein cholesterol (HDL) 54
    mg/dL, triglycerides 145 mg/dL, total testosterone
    concentration 970 ng/dL (reference range 270–
    1070 ng/dL), and free testosterone concentration
    22 ng/dL (reference range 9–30 ng/ dL). Which
    drug is best to initiate for his erectile dysfunction?
A
  1. Answer: A
    The patient’s laboratory values are within normal limits,
    indicating that his disease states are not necessarily the
    cause of his erectile dysfunction. A phosphodiesterase
    inhibitor such as vardenafil may be initiated (Answer
    A is correct). Testosterone replacement would not be
    effective because the patient has normal testosterone
    concentrations (Answer B is incorrect). Yohimbine
    would not be considered first-line therapy because
    its efficacy is controversial (Answer C is incorrect).
    Fluoxetine is inappropriate because the patient does not
    have premature ejaculation (Answer D is incorrect).
How well did you know this?
1
Not at all
2
3
4
5
Perfectly
12
Q
  1. T.M., a 33-year-old man, has a history of intravenous
    drug abuse and lives in and out of homeless
    shelters. He is taken to the emergency department
    by ambulance after experiencing paralysis on the
    right side of his body. The people at the shelter
    thought he might be having a stroke. In the emergency
    department, screening was positive for the
    Venereal Disease Research Laboratory test (syphilis
    test) with 10 white blood cells/mm3. T.M. has
    no known significant medical history (except for
    treatment of a sexually transmitted disease [STD]),
    but he is allergic to penicillin (anaphylactic reaction).
    Assuming serologic testing confirms a diagnosis
    of syphilis, which therapy is best for T.M.?
    A. Levofloxacin 750 mg intravenously for one
    dose.
    B. Penicillin G 4 million units every 4 hours
    intravenously for 14 days after penicillin
    desensitization.
    C. Benzathine penicillin G 2.4 million units
    intramuscularly every week for 3 weeks after
    penicillin desensitization.
    D. Azithromycin 500 mg intravenously or orally
    daily for 6 weeks.
A
  1. Answer: B
    Penicillin G 4 million units every 4 hours intravenously
    for 14 days after penicillin desensitization is the correct
    therapy for a patient with neurosyphilis who is allergic
    to penicillin (Answer B is correct). Levofloxacin would
    not cover syphilis (Answer A is incorrect). Three doses
    of benzathine penicillin G are indicated for late latent
    syphilis, not neurosyphilis (Answer C is incorrect).
    Furthermore, although azithromycin is an alternative
    for patients who are penicillin allergic in other situations,
    patients with neurosyphilis should be desensitized
    and given penicillin (Answer D is incorrect).
How well did you know this?
1
Not at all
2
3
4
5
Perfectly
13
Q
  1. A prospective double-blind study compared the
    effects of three different antivirals—acyclovir,
    famciclovir, and valacyclovir—in 360 patients
    with first-episode genital herpes. Which statistical
    test is best to compare the mean duration of time
    until the lesions healed?
    A. Analysis of variance (ANOVA).
    B. Chi-square test.
    C. Mann-Whitney U test.
    D. Student t-test.
A
  1. Answer: A
    Data are continuous and probably normally distributed
    (given the large population of 360 patients in
    the study); therefore, a parametric test is indicated.
    Because ANOVA is a parametric test used to compare
    more than two groups, it would be appropriate (Answer
    A is correct). The Student t-test is a parametric test for
    comparing only two groups (Answer D is incorrect). A
    chi-square test is used to assess nominal data between
    two groups (Answer B is incorrect). The Mann-Whitney
    U test is a nonparametric analog to the Student t-test
    (Answer C is incorrect).
How well did you know this?
1
Not at all
2
3
4
5
Perfectly
14
Q

Patient Case
Questions 1 and 2 pertain to the following case:
E.L. is a 50-year-old woman with hot flashes and vaginal irritation. She has tried exercise, diet, and antidepressants
to help relieve her hot flashes but has been unsuccessful. She is otherwise healthy with no history of cancer and
no surgical procedures. She states that her hot flashes are interfering with her daily activities and wants to try HT.
1. When counseling E.L. on the use of HT and explaining the WHI trial, which has been proven to be statistically
significant with conjugated estrogen and medroxyprogesterone acetate?
A. Increased risk of VTE
B. Decreased risk of stroke
C. Decreased risk of MI
D. Increased risk of fractures

A
  1. Answer: A
    DVT is increased with conjugated estrogens and
    medroxyprogesterone acetate (Prempro) (Answer A
    is correct). Myocardial infarction and strokes are also
    increased (Answers B and C are incorrect). Fractures
    are decreased (Answer D is incorrect).
How well did you know this?
1
Not at all
2
3
4
5
Perfectly
15
Q

E.L. is a 50-year-old woman with hot flashes and vaginal irritation. She has tried exercise, diet, and antidepressants
to help relieve her hot flashes but has been unsuccessful. She is otherwise healthy with no history of cancer and
no surgical procedures. She states that her hot flashes are interfering with her daily activities and wants to try HT.

  1. Which treatment is best to recommend to E.L.?
    A. Estrogen patch 0.025 mg (17β-estradiol); change patch twice weekly.
    B. Prasterone 6.5 mg vaginal inserts; insert vaginally once daily.
    C. Conjugated estrogen 0.3 mg/medroxyprogesterone acetate 1.5 mg; take 1 tablet daily.
    D. Ospemifene 60 mg; take 1 tablet daily.
A
  1. Answer: C
    The patient has an intact uterus; therefore, she needs
    both an estrogen and a progestogen. Conjugated estrogens
    and medroxyprogesterone acetate (Prempro) is the
    only product listed that includes a progestogen (Answer
    C is correct). The patient has hot flashes in addition to
    vaginal dryness; therefore, a systemic product is recommended
    (Answer B is incorrect). A vaginal insert or
    cream would be appropriate for GSM symptoms but not
    for hot flashes, and ospemifene is indicated for vaginal
    atrophy. The patch would be appropriate if a progestogen
    were added to the regimen (Answers A and D are
    incorrect).
How well did you know this?
1
Not at all
2
3
4
5
Perfectly
16
Q

R.K. is a 71-year-old woman (height 63 inches, weight 64 kg) with a history of rheumatoid arthritis who smokes
½ pack/day. She is lactose intolerant and has minimal intake of dairy products. She takes calcium 1200 mg orally
per day in divided doses and vitamin D 600 international units/day orally. Her calculated CrCl is 60–70 mL/minute.
Her BMD T-score is -2.6 at the hip and -2.1 at the spine. Her FRAX score indicates she has a 10-year probability of
a major osteoporotic fracture of 22% and a 10-year probability of a hip fracture of 11%.
3. Which statement best describes the correct diagnosis for R.K.?
A. She has normal BMD of the spine.
B. She has low bone mass (osteopenia) of the hip.
C. She has osteoporosis of the hip.
D. She has severe osteoporosis of the spine.

A
  1. Answer: C
    The definitions of bone mineral density and T-scores
    are as follows:
    Normal = BMD within 1 standard deviation (SD) of the
    young adult mean, Low bone mass (osteopenia) = BMD
    between -1 SD and -2.5 SD,
    Osteoporosis = BMD at least -2.5 SD,
    Severe osteoporosis = BMD less than -2.5 and history
    of a fracture. The patient has a T-score of -2.6 at the
    hip, which is indicative of osteoporosis (Answer C is
    correct; Answers A and B are incorrect). The patient
    does not have severe osteoporosis of the spine because
    her T-score at the spine is -2.1 and not less than -2.5
    (Answer D is incorrect).
How well did you know this?
1
Not at all
2
3
4
5
Perfectly
17
Q

R.K. is a 71-year-old woman (height 63 inches, weight 64 kg) with a history of rheumatoid arthritis who smokes
½ pack/day. She is lactose intolerant and has minimal intake of dairy products. She takes calcium 1200 mg orally
per day in divided doses and vitamin D 600 international units/day orally. Her calculated CrCl is 60–70 mL/minute.
Her BMD T-score is -2.6 at the hip and -2.1 at the spine. Her FRAX score indicates she has a 10-year probability of
a major osteoporotic fracture of 22% and a 10-year probability of a hip fracture of 11%.

  1. Which is the best therapy for R.K.?
    A. No further treatment is needed; continue calcium 1200 mg/vitamin D 600 international units/day orally.
    B. Give abaloparatide 80 mcg subcutaneously daily, and continue calcium 1200 mg/vitamin D 600 international
    units/day orally.
    C. Give romosozumab 210 mg subcutaneously every month, continue calcium 1200 mg/day orally, and
    increase vitamin D to 800 international units/day orally.
    D. Give risedronate 35 mg orally every week; continue calcium 1200 mg orally per day, and increase vitamin
    D to 800 international units/day orally.
A
  1. Answer: D
    Drug therapy is indicated for the following: A hip or
    vertebral fracture; a BMD T-score below -2.5 at the
    femoral neck or spine, excluding secondary causes; or
    a BMD T-score between -1.0 and -2.5 at the femoral
    neck or spine and a 10-year probability of hip fracture
    3% or greater or a 10-year probability of major osteoporosis-
    related fracture of 20% or greater based on the
    FRAX system. The patient needs therapy because her T-score is less than -2.5 at the hip, and her 10-year probability
    of hip fracture is at least 3% or her 10-year probability
    of major osteoporosis-related fracture at least
    20% based on the FRAX system (Answer D is correct;
    Answer A is incorrect). Bisphosphonates such as alendronate
    and risedronate are considered first-line drugs
    because they inhibit normal and abnormal bone resorption
    and reduce vertebral and non-vertebral fractures
    by 30%–50%. For risendronate, the treatment dosage is
    35 mg orally weekly, further indicating that Answer D
    is correct. In addition, the patient’s CrCl is 60–70 mL/
    minute, which allows use of a bisphosphonate. Use of
    a bisphosphonate is contraindicated at a CrCl less than
    30–35 mL/minute. Abaloparatide (Tymlos) is reserved
    for treating women at a high risk of fracture, including
    those with a very low BMD (T-score less than -3.0) and
    a previous vertebral fracture (Answer B is incorrect).
    The patient should receive vitamin D 800 international
    units daily because she is older than 70 years and takes
    calcium 1200 mg daily in divided doses (again, Answer
    D is correct; Answers A and B are incorrect). In addition,
    the patient is likely not receiving calcium from her
    diet due to her lactose intolerance and minimal intake
    of dairy products. Romosozumab is not indicated as
    first-line treatment and is indicated for those at high risk
    of fracture, those with a previous fracture, or those for
    whom other osteoporosis therapies have failed (Answer
    C is incorrect).
How well did you know this?
1
Not at all
2
3
4
5
Perfectly
18
Q
  1. S.E. is a 28-year-old woman who would like to get pregnant soon. Her medical history includes hypertension
    and allergies. Her medications include lisinopril, nasal saline spray, and folic acid. Which option is best to treat
    her hypertension while she is pregnant or trying to conceive?
    A. Continue lisinopril and add hydrochlorothiazide.
    B. Discontinue lisinopril and all other medications.
    C. Discontinue lisinopril and start labetalol.
    D. Continue lisinopril and add atenolol.
A
  1. Answer: C
    Lisinopril is an ACE inhibitor known to have some
    teratogenicity. It is not recommended for women who
    are trying to conceive or who are pregnant (Answers
    A and D are incorrect). This patient needs treatment
    for hypertension, particularly while pregnant, because
    hypertension can lead to detrimental effects in the
    fetus and mother (Answer B is incorrect). Labetalol
    and nifedipine are the preferred agents for treating
    hypertension in women who are trying to conceive or
    pregnant, with labetalol used more often in recent years
    (Answer C is correct). β-Blockers are not specifically
    teratogenic but may cause adverse effects in the fetus
    such as intrauterine growth retardation, particularly
    with atenolol (Answer D is incorrect). Diuretics such
    as hydrochlorothiazide are not first line. They can be
    used as a second- or third-line agent. (again, Answer A
    is incorrect).
19
Q
  1. A 22-year-old woman started taking levonorgestrel 0.1 mg/ethinyl estradiol 20 mcg tablet daily 4 months ago
    for contraception. She has breakthrough bleeding at the start of her active pills that lasts a few days before
    resolving. As a pharmacist working under a collaborative practice agreement, which OC on her formulary is
    best to prescribe?

Name of OC Estrogen

Levonorgestrel 0.1 mg/ethinyl estradiol 20 mcg =
Desogestrel 0.15 mg/ethinyl estradiol 30 mcg
Norethindrone 0.5 mg/ethinyl estradiol 35 mcg
Norethindrone acetate 1.5 mg/ethinyl estradiol 30 mcg

A. Continue levonorgestrel 0.1 mg/ethinyl estradiol 20 mcg for another 3 months.
B. Change to desogestrel 0.15 mg/ethinyl estradiol 30 mcg.
C. Change to norethindrone acetate 1.5 mg/ethinyl estradiol 30 mcg.
D. Change to norethindrone 0.5 mg/ethinyl estradiol 35 mcg.

A
  1. Answer: B
    Estrogen deficiency causes early or midcycle breakthrough
    bleeding (days 1–14). There is possible deficiency
    in estrogen and progestin at mid-cycle. Progestin
    deficiency results in late breakthrough bleeding (days
    14–28). An OC with stronger estrogenic properties is
    needed because the patient is bleeding through early in
    the cycle. Ethinyl estradiol 30 mcg/desogestrel 0.3 mg
    (Reclipsen) has higher estrogenic properties (intermediate)
    than ethinyl estradiol 20 mg/levogestrel 0.1 mg
    (Lessina) while the other OC options have low estrogenic
    properties (Answer B is correct; Answers C and D
    are incorrect). The patient has been using Lessina for 4
    months, which is past the trial period of 3 months, with
    continued breakthrough bleeding. Therefore, the patient
    could switch to another product (Answer A is incorrect).
20
Q
  1. L.M., a 37-year-old woman (height 67 inches, weight 95 kg), states that she wants to begin contraception for
    now; however, she would like to have children in a year or so. Her medical history includes hypertension
    for 2 years and gastroesophageal reflux disease; she drinks 2 glasses of wine a week and smokes ½ pack of
    cigarettes/day. Her medications include hydrochlorothiazide 25 mg orally daily, amlodipine 5 mg /benazepril
    20 mg orally daily, omeprazole 20 mg orally daily, and occasional ibuprofen. Which contraceptive product is
    best to recommend for L.M.?
    A. Transdermal contraceptive patch
    B. Oral tablet ethinyl estradiol/drospirenone
    C. Oral tablet norethindrone
    D. DMPA IM injection
A
  1. Answer: C
    Estrogen-containing products (ethinyl estradiol/dropsirenone
    and contraceptive patch) are contraindicated
    for this patient because she is a smoker older than 35.
    In addition, the patient cannot use the contraceptive
    patch (ethinyl estradiol and norelgestromin [Xulane])
    because her weight is more than 90 kg. Even though
    she smokes only ½ pack of cigarettes per day, her other
    risk factors such as age and hypertension increase the
    risk of events with tobacco use. In addition, she takes
    an ACE inhibitor, which would not be the best choice
    with drospirenone because of the possibility of hyperkalemia
    (Answer B is incorrect). Depot medroxyprogesterone
    acetate has a long return to fertility time,
    and the patient is 37 years old (Answer D is incorrect).
    Moreover, DMPA may cause weight gain, and the
    patient already has obesity. Norethindrone has a quick
    return to fertility and will not affect her hypertension or
    interact with her ACE inhibitor (Answer C is correct).
21
Q
  1. L.L. is a 26-year-old woman (height 61 inches, weight 78 kg) who has been trying to conceive for 13 months
    without success. She and her male partner would like to conceive in the next year or so. Her BMI is 32 kg/m2, and
    she has moderate acne and hirsutism. Her menstrual cycle is fairly regular at 26–27 days. Her partner’s physical
    examination and semen analysis are normal. Her pelvic ultrasonography reveals polycystic ovaries, and she is
    given a diagnosis of PCOS. Which is the best first-line agent to recommend to L.L. to help her conceive?
    A. Consider weight loss and start clomiphene tablets.
    B. Continue trying to conceive; make no recommendations at this time.
    C. Start FSH injections.
    D. Start hCG injections.
A
  1. Answer: A
    The patient has obesity, with a BMI of 32 kg/m2. To
    optimize her chances of ovulation, she should try
    to lose weight and lower her BMI. Her ultrasonography,
    hirsutism, and acne indicate she has PCOS.
    Clomiphene citrate would be a first-line agent to help
    stimulate ovulation (Answer A is correct). She and her
    husband have been trying to conceive for 13 months,
    which meets the definition of infertility necessitating
    intervention (Answer B is incorrect). Human chorionic
    gonadotropin injections are not appropriate to
    provide at this time; they are usually used to help the ovum rupture once the follicle has been stimulated to
    increase (Answer D is incorrect). Follicle-stimulating
    hormone would not be recommended at this time as it
    is not a first-line treatment (Answer C is incorrect). If
    the patient fails clomiphene or letrozole (another option
    instead of clomiphene but not an option in the answers)
    then FSH injections might be appropriate at that time.
22
Q

D.H. is a 21-year-old woman who presents to the clinic with genital itching and vesicles on her vulva. She is sexually
active with one partner who has a history of herpes. Her partner does not always use a condom when they have
sex. She is initiated on acyclovir for this initial HSV infection.
9. Which statement is best to mention to D.H. regarding the treatment of her herpes infection?
A. Treatment of the initial infection will prevent recurrent herpes infections.
B. Treatment will shorten the duration of symptoms and infectivity of the initial infection.
C. Treatment of the initial infection will decrease the severity of recurrent herpes infections.
D. Treatment of the initial infection will prevent the virus from remaining latent in the dorsal root ganglia.

A
  1. Answer: B
    Treatment of HSV infection substantially decreases the
    duration of viral shedding, pain, and time to complete
    healing but does not affect the risk, frequency, latency,
    or severity of recurrences (Answer B is correct; all
    others are incorrect).
23
Q

D.H. is a 21-year-old woman who presents to the clinic with genital itching and vesicles on her vulva. She is sexually
active with one partner who has a history of herpes. Her partner does not always use a condom when they have
sex. She is initiated on acyclovir for this initial HSV infection.

  1. D.H. returns to the clinic 10 months after her initial herpes infection. She is troubled by all the recurrences
    she is having (seven to date). Which therapy is best to recommend?
    A. Valacyclovir 500 mg orally twice daily to be used for 5 days whenever she notices a recurrence
    beginning.
    B. Acyclovir 400 mg orally three times daily to be used for 10 days whenever she notices a recurrence
    beginning.
    C. Suppressive therapy with famciclovir 250 mg orally three times daily.
    D. Suppressive therapy with valacyclovir 500 mg daily orally.
A
  1. Answer: D
    Patient-initiated therapy is important for people with
    occasional recurrences (fewer than six per year) of
    HSV infection because recurrent infections resolve
    more rapidly than the initial infection. Antiviral agents
    should be initiated as soon as possible. Because the
    patient is having several recurrences (six or more a
    year), patient-initiated therapy is insufficient (Answers
    A and B are incorrect). Suppressive therapy is given
    twice daily or once daily, not three times daily; therefore,
    suppressive therapy with famciclovir 250 mg
    orally three times daily is inappropriate (Answer C is
    incorrect). Suppressive therapy with valacyclovir 500
    mg/day orally should be offered (Answer D is correct).
24
Q

M.A. is a 24-year-old woman who presents to the emergency department with severe abdominal pain, fever,
dysuria, and a vaginal discharge. She is sexually active. Her medical history is unremarkable except for recurrent
genital herpes (one or two episodes a year). Her medications on admission include birth control pills (ethinyl
estradiol 30 mcg/desogestrel 0.15 mg) and fluticasone nasal spray as needed. On physical examination, M.A.’s vital
signs include temperature 101.2°F (38°C), heart rate 92 beats/minute, respiration rate 15 breaths/minute, and blood
pressure 117/75 mm Hg. M.A. has adnexal tenderness, cervical motion tenderness, and a vaginal discharge.
11. Which is the best empiric therapy for M.A.?
A. Ampicillin/sulbactam intravenously 2 g every 6 hours for 14 days
B. Metronidazole 500 mg intravenously three times daily for 7 days
C. Cefotetan 2 g intravenously every 12 hours with doxycycline 100 mg orally every 12 hours for 14 days
D. Ceftriaxone 125 mg intramuscularly once with doxycycline 100 mg intravenously twice daily for 7 days

A
  1. Answer: C
    Cefotetan 2 g intravenously every 12 hours with doxycycline
    100 mg orally or intravenously every 12 hours
    is an appropriate empiric antibiotic combination for
    PID (Answer C is correct). The combination has activity
    against N. gonorrhoeae and C. trachomatis and
    against the gram-negative and anaerobic organisms
    that are often involved. Metronidazole alone would
    have activity only against anaerobes and would miss
    the other organisms often involved in PID (Answer B
    is incorrect). Ampicillin/sulbactam has good activity
    against most organisms in PID; however, it has no activity
    against atypical organisms (e.g., C. trachomatis)
    (Answer A is incorrect). Ceftriaxone and doxycycline
    are likely not sufficient for empiric coverage of PID as
    there is not anaerobic coverage. Additionally, the ceftriaxone
    dosage should be 1 g, the duration of doxycycline
    should be 14 days, and metronidazole should be a component
    of the regimen (Answer D is incorrect).
25
Q

M.A. is a 24-year-old woman who presents to the emergency department with severe abdominal pain, fever,
dysuria, and a vaginal discharge. She is sexually active. Her medical history is unremarkable except for recurrent
genital herpes (one or two episodes a year). Her medications on admission include birth control pills (ethinyl
estradiol 30 mcg/desogestrel 0.15 mg) and fluticasone nasal spray as needed. On physical examination, M.A.’s vital
signs include temperature 101.2°F (38°C), heart rate 92 beats/minute, respiration rate 15 breaths/minute, and blood
pressure 117/75 mm Hg. M.A. has adnexal tenderness, cervical motion tenderness, and a vaginal discharge.

  1. Which statement is best for M.A. to tell her sexual partners?
    A. There is no need for concern because this condition is not transmittable to or acquired from a sexual
    partner.
    B. Her partner can resume having sexual intercourse with M.A. as soon as her symptoms improve.
    C. If her partner has had sex with M.A. within the past 60 days, he should be assessed for possible treatment.
    D. Her partner does not need to be tested for human immunodeficiency virus (HIV) because there is no
    relationship between HIV and this condition.
A
  1. Answer: C
    Pelvic inflammatory disease is related to sexual activity;
    therefore, until all partners and the patient have been
    treated, abstinence from intercourse for at least 7 days
    is indicated, (Answers A and B are incorrect). Patients
    should be encouraged to be tested for HIV because of
    the strong relationship between STDs and the risk of
    HIV (Answer D is incorrect). The most appropriate
    recommendation for a sexually active patient with PID
    is to have all sexual partners within the past 60 days
    tested and treated (Answer C is correct).
26
Q
A
  1. Answer: A
    Tadalafil may be dosed daily without respect to timing
    of sexual intercourse (Answer A is correct). Avanafil
    should be taken 30 minutes before sexual intercourse
    (Answer B is incorrect). Yohimbine and bupropion are
    not first line for erectile dysfunction (Answers C and
    D are incorrect). Bupropion is usually used in women
    with sexual dysfunction caused by serotonin reuptake
    inhibitors and not usually used in men for the treatment
    of sexual dysfunction.
27
Q

Questions 1 and 2 pertain to the following case.
A 36-year-old woman is in the clinic for her 2-week postpartum
checkup, wanting to know which contraceptive
method she should use. She had to stop breastfeeding when
she was 5 days postpartum, after she had a stroke. Her
medical history is significant for morbid obesity, a tilted
and bicornate uterus, allergic rhinitis, and a cerebrovascular
accident (5 days postpartum). She is allergic to latex.
Current medications are lisinopril 5 mg/day, hydrochlorothiazide

  1. Which is the best contraceptive recommendation for
    this woman?
    A. Depot medroxyprogesterone acetate.
    B. Levonorgestrel intrauterine device (LNG-IUD).
    C. Contraceptive sponge.
    D. Male polyurethane condom
    12.5 mg/day, simvastatin 20 mg every night, and
    aspirin 81 mg/day (all medications initiated 1.5 weeks ago).
A
  1. Answer: D
    Depot medroxyprogesterone acetate (Answer A) causes
    considerable weight gain, making it a less-than-optimal
    choice for this patient. Cerebrovascular accident is also
    a relative contraindication (medical eligibility criteria
    category 3) for progestin-only contraceptives of depot
    medroxyprogesterone and continuation of POPs and
    implants. The levonorgestrel IUD (Answer B) should not
    be used because the patient has structural abnormalities of
    the uterus. The contraceptive sponge (Answer C) should
    be avoided because of the patient’s uterine structural
    abnormalities and because she is recently postpartum.
    The polyurethane condom (Answer D) is the best option
    because the patient has no contraindications or allergies
    that would prohibit its use.
28
Q

Questions 1 and 2 pertain to the following case.
A 36-year-old woman is in the clinic for her 2-week postpartum
checkup, wanting to know which contraceptive
method she should use. She had to stop breastfeeding when
she was 5 days postpartum, after she had a stroke. Her
medical history is significant for morbid obesity, a tilted
and bicornate uterus, allergic rhinitis, and a cerebrovascular
accident (5 days postpartum). She is allergic to latex.
Current medications are lisinopril 5 mg/day, hydrochlorothiazide
12.5 mg/day, simvastatin 20 mg every night, and
aspirin 81 mg/day (all medications initiated 1.5 weeks ago).

  1. The patient calls to ask for another contraceptive
    choice because she cannot afford the item you recommended.
    She states that the free clinic does not carry
    the item either. Of the alternative contraceptives that
    can be provided free from either your clinic or the free
    clinic, which is the best recommendation?
    A. Female condom.
    B. Male latex condom.
    C. Yaz (ethinyl estradiol and drospirenone).
    D. Ella (ulipristal).
A
  1. Answer: A
    The patient has no contraindications or allergies to the
    female condom (Answer A). However, she has a latex
    allergy, so the male latex condom (Answer B) would not
    be a good choice. Estrogen-containing contraceptives
    (Answer C) are contraindicated (medical eligibility criteria
    category 4) for patients with stroke, according to the CDC
    and the World Health Organization. Ulipristal (Answer D)
    is a form of EC; it should not be used as a regular form of
    contraception.
29
Q
  1. A double-blind randomized trial is under way to
    evaluate the effects of depot medroxyprogesterone,
    leuprolide, and placebo on the bone mineral density of
    600 patients with endometriosis. Which statistical test
    is most appropriate?
    A. Student t test.
    B. Fisher exact test.
    C. Kruskal-Wallis test.
    D. Analysis of variance.
A
  1. Answer: D
    Analysis of variance (Answer D) would be most appropriate
    because the trial consists of more than two groups and
    involves continuous data that are most likely normally distributed
    (n=600). Although the Student t test (Answer A)
    is for continuous data, it should be used only if two groups
    are being compared. Both the Fisher exact test (Answer B)
    and the Kruskal-Wallis test (Answer C) are for nonparametric
    data.
30
Q
  1. A 40-year-old woman asks to see the pharmacist after
    her physician’s appointment. She states that she was
    prescribed a new drug during her pregnancy. She is
    uncomfortable taking medications during her pregnancy
    because her family said that they all carry risk.
    Which is the best information to include when educating
    the patient on the risks and benefits of the drug?
    A. Rate of birth defects in studies of animals.
    B. Gestational timing of risks and pregnancy.
    C. Molecular weight of the drug.
    D. Half-life of the medication.
A
  1. Answer: B
    Although molecular weight is important in determining
    whether a drug will cross the placenta, the risk of malformations
    is not directly addressed (Answer C is incorrect).
    Information from studies of animals helps determine the
    potential risks of drugs in humans but does not confer
    exact risks in humans (Answer A is incorrect). Educating
    the patient on the gestational timing of risks and on the
    current stage of pregnancy is imperative to understanding
    whether the patient has any chance of experiencing that birth defect (Answer B is correct). The half-life of a
    medication does not affect the risk of medication exposure
    during pregnancy (Answer D is incorrect).
31
Q
  1. A 32-year-old woman who is 4 weeks postpartum calls
    your office asking whether it is okay for her to start
    terbinafine therapy for 6 months for toe onychomycosis
    that began during the pregnancy. She states that
    she consulted a podiatrist yesterday and that the podiatrist
    gave her this prescription. She reports no pain,
    redness, or difficulty walking but states she does not
    like how her toes look when wearing sandals. She is currently breastfeeding every 2 hours. You will find
    the following information regarding use in breastfeeding
    in the reference Medications & Mothers’ Milk
    (Hale 2019): milk/plasma ratio unknown; relative
    infant dose unknown; half-life 26 hours; 99% protein
    bound; and molecular weight 291 Da. Which is the
    best recommendation?
    A. Delay treatment until finished with breastfeeding.
    B. Change to itraconazole.
    C. Use topical terbinafine.
    D. Schedule doses right after feedings.
A
  1. Answer: A
    The best recommendation is to delay the treatment until
    after she stops breastfeeding (Answer A is correct). The
    drug is likely to cross into breast milk, but exact concentrations
    are unknown. Because of its long half-life and therapy
    duration, the infant would be exposed to the drug. The
    patient, who is currently asymptomatic, is seeking treatment
    only for cosmetic reasons. Itraconazole is an option
    for treating onychomycosis; however, it has decreased efficacy
    compared with terbinafine (Answer B is incorrect).
    Topical terbinafine is not effective for treating onychomycosis
    (Answer C is incorrect). Scheduling the doses right
    after feedings is recommended to minimize infant exposure;
    however, because the half-life is long and the baby is
    feeding every 2 hours, this recommendation is unlikely to
    decrease infant exposure (Answer D is incorrect).
32
Q
  1. A 21-year-old woman is in the office for a follow-up of
    her dysmenorrhea. She states that because ibuprofen
    has only slightly improved her pain, she would like
    something else. She is currently in a monogamous
    relationship and would like contraceptive protection
    as well. Her vital signs today include the following:
    height 63 inches, weight 99 kg (220 lb), blood pressure
    118/68 mm Hg, and heart rate 72 beats/minute.
    Which is the best recommendation?
    A. Ethinyl estradiol and norelgestromin (Xulane):
    Apply one patch every week for 3 weeks; then
    repeat after a 1-week hormone-free interval.
    B. Ethinyl estradiol and norelgestromin (Xulane):
    Apply one patch every week for 11 weeks; then
    repeat after a 1-week hormone-free interval.
    C. Ethinyl estradiol 35 mcg and ethynodiol diacetate
    1 mg (Kelnor 1/35): Take one tablet every day for
    3 weeks; then repeat after a 7-day hormone-free
    interval.
    D. Ethinyl estradiol 35 mcg and ethynodiol diacetate
    1 mg (Kelnor 1/35): Take one tablet every day for
    11 weeks; then repeat after a 7-day hormone-free
    interval.
A
  1. Answer: D
    Because the patient has a BMI >30, Xulane is not recommended
    because of decreased efficacy and increased
    thromboembolic risk, making Answer A and Answer B
    incorrect. Estrogen-progestin contraceptives (Answer C
    and Answer D) are second-line agents after NSAIDs for
    treating dysmenorrhea because they can decrease menstrual
    length and volume. Extended-interval dosing is
    preferred because it decreases the frequency of menses,
    making Answer D correct.
33
Q
  1. A 49-year-old woman is initiating therapy with
    estradiol valerate and dienogest (Natazia) for perimenopausal
    symptoms and contraceptive needs. You
    are asked to educate her about this product. Which
    option provides the best information for the patient
    regarding the minimal time a backup method of contraception
    should be used after initiation?
    A. 48 hours.
    B. 7 days.
    C. 9 days.
    D. 28 days.
A
  1. Answer: C
    Natazia is a quadriphasic hormonal contraceptive that
    requires 9 days of backup contraception. The first two pills
    contain only estrogen, and ovulation protection does not
    occur until after the seventh dose. The egg is only viable
    for up to 48 hours after ovulation, so 9 days (Answer C) are
    required to provide pregnancy protection.
34
Q
  1. A 38-year-old woman is calling because of the intolerable
    vasomotor symptoms she is experiencing, which
    interfere with her daily activities. Her medical history
    includes breast cancer (diagnosed 3 months ago).
    She takes trastuzumab. She notes that the physician
    told her that her treatment caused menopause. She
    states her hot flashes occur at least 12 times a day
    and cause her to change clothes often. She would like
    additional therapy. Blood pressure is 104/64 mm Hg,
    and heart rate is 66 beats/minute. Which is the best
    recommendation?
    A. Conjugated equine estrogens.
    B. Venlafaxine.
    C. Clonidine.
    D. Black cohosh.
A
  1. Answer: B
    Estrogens should be avoided because of the patient’s
    active breast cancer (Answer A is incorrect). Venlafaxine
    has shown efficacy in decreasing vasomotor symptoms in
    patients with and without breast cancer (Answer B is correct).
    Clonidine improves vasomotor symptoms but may
    not be the best choice because this patient’s blood pressure is low (Answer C is incorrect). Black cohosh has not been
    effective in reducing vasomotor symptoms in patients with
    breast cancer (Answer D is incorrect).
35
Q
  1. A 25-year-old woman was recently given a diagnosis
    of endometriosis. She is having trouble coping with
    the diagnosis and wants to find a support group. Which
    is the best resource for finding local support groups?
    A. Association of Reproductive Health Professionals.
    B. American College of Obstetricians and
    Gynecologists (ACOG).
    C. Endometriosis Association.
    D. National Women’s Health Network.
A
  1. Answer: C
    The Endometriosis Association (Answer C) provides
    contact information for local support groups and patient
    information. The Association of Reproductive Health
    Professionals (Answer A), ACOG (Answer B), and the
    National Women’s Health Network (Answer D) provide
    patient information and health care–related information.
36
Q
  1. A 39-year-old woman is requesting hormonal contraception. She plans to start attempting conception in about 12
    months. The woman is currently 6 weeks postpartum, and she is feeding formula to the infant. Her medical history
    is significant for gestational diabetes, hypertension, and hyperthyroidism. She states she is concerned about
    losing her pregnancy weight. Current medications are propylthiouracil 100 mg three times daily, lisinopril 10 mg/
    day, hydrochlorothiazide 25 mg/day, and a prenatal vitamin one tablet daily. With respect to social history, the
    patient reports no tobacco use, uses ethanol socially, and reports not using illegal drugs. Her height is 65 inches;
    her weight today is 131 kg (290 lb), BMI 48 kg/m2 (pre-pregnancy weight of 104 kg [230 lb]), BMI 38 kg/m2.
    Blood pressure is 178/96 mm Hg today (188/102 mm Hg 2 weeks ago). Which is the most appropriate hormonal
    contraceptive recommendation?
    A. Depo-Provera (medroxyprogesterone acetate).
    B. Xulane (ethinyl estradiol and norelgestromin).
    C. Yaz (ethinyl estradiol and drospirenone).
    D. Micronor (norethindrone).
A
  1. Answer: D
    This patient currently has contraindications to using an
    estrogen-containing contraceptive because her systolic
    blood pressure is greater than 160 mm Hg (Answers B and
    C are incorrect). Medroxyprogesterone acetate (Answer A)
    would not be the best choice because the patient is already
    overweight, and this product causes significant weight gain.
    In addition, medroxyprogesterone has an average return of
    fertility of 9 months after the last injection. Because the
    patient wants to attempt conception in 12 months, she
    would receive only a few injections and would then need
    to consider alternative agents. Norethindrone (Answer D)
    does not contain estrogen and is not associated with significant
    weight gain.
37
Q
  1. A 21-year-old woman has been taking contraceptive X for the past 8 months. She calls today because she had been
    experiencing breakthrough bleeding for 2 days, and then her menses began 4–5 days later. She states it is bothersome
    to have so much bleeding in the past two cycles. Her medical history includes dysmenorrhea.
A
  1. Answer: A
    Late-cycle breakthrough bleeding is a result of progestin
    deficiency. The patient must start using a different product
    that has higher progestin activity (+++) than her current
    product (++) (Answer A). Answers B, C, and D do not
    increase progestin activity, so these are incorrect.
38
Q
  1. A 17-year-old patient is crying in the examination room, saying she does not know what to do. She tells you that
    she had chlamydia a few months ago and was shocked, so she did not have sex again until a few days ago with her
    new boyfriend. Because she did not want to have chlamydia again, they used male and female condoms, but both
    broke. She is afraid she has chlamydia again because of vaginal discharge and fears getting pregnant. She wants
    to know whether her tests are back and what she can use for EC. Her laboratory tests just resulted in the electronic
    medical system and were positive for chlamydia and trichomoniasis. Her medical history includes chlamydia (4
    months ago) and acne. After discussing it with you, her provider plans to prescribe doxycycline 100 mg twice
    daily for 7 days and metronidazole 500 mg twice daily for 7 days today. Her current medications include OTC
    adapalene at bedtime. Which is the best recommendation?
    A. Recommend Plan B One-Step (levonorgestrel 1.5 mg) every 12 hours for two doses.
    B. Recommend a levonorgestrel IUD today.
    C. Recommend ulipristal 30 mg for one dose.
    D. Refuse EC because it has been too long since unprotected intercourse.
A
  1. Answer: C
    Answer A is incorrect because this patient is past the
    optimal dosing window of 72 hours for Plan B One-Step.
    Studies have shown continued efficacy for up to 120
    hours after unprotected intercourse, but efficacy may be
    decreased when used past 72 hours. The dosing in Answer
    A is also incorrect. Dosing should be levonorgestrel 1.5 mg
    for one dose. Answer B, recommend a levonorgestrel IUD
    today, is incorrect because she has an active chlamydia
    infection. Medical eligibility criteria state that initiating
    an IUD when the patient has a current purulent cervicitis,
    chlamydia, or gonococcal infection is a category 4 rating;
    hence, this is contraindicated. Ulipristal (Answer C) can be
    given up to 120 hours after unprotected intercourse and can
    be initiated immediately. Answer D is incorrect because
    she can still receive EC, given that it has been fewer than
    120 hours since unprotected intercourse.
39
Q
  1. A 42-year-old woman was referred to the office by her local pharmacist because her heavy bleeding was not
    improving with ibuprofen. She says that her menstrual cycles have always been heavy but that now, they seem
    excessive and are coming every 2.5–3 weeks. She admits feeling tired and having shortness of breath with longer
    walks. She admits craving corn starch and eating a jar over 3 days. She states that she would be willing to use
    a contraceptive. Her blood pressure today was 130/72 mm Hg. Her BMI is 29 kg/m2. Her laboratory test results
    today show hemoglobin 9 g/dL, hematocrit 28%, mean corpuscular volume 77 fL/red cell, red cell distribution
    width 14%, platelet count 180,000/mm3, percent iron saturation 2%, and ferritin 5 ng/mL. Her comprehensive
    metabolic panel and thyroid stimulating hormone concentrations were normal. Her hCG was negative. Her medical
    history is significant for dysmenorrhea, DVT (occurred when 20 years old), and uterine fibroids. Her current
    medication is ibuprofen 600 mg every 6 hours as needed for pain (uses during each menses). Which is the best
    recommendation?
    A. LNG-IUD (Liletta).
    B. Estrogen/progestin pill (Portia).
    C. Estrogen-progestin vaginal ring (NuvaRing).
    D. Tranexamic acid (Lysteda).
A
  1. Answer: A
    This patient has heavy menstrual bleeding, and her NSAID
    therapy has failed. If patients are interested in using a
    contraceptive, LNG-IUD have the highest efficacy in
    improving symptoms, Answer A. Answers B and C are
    second-line options but are contraindicated in this patient
    because of her history of deep venous thrombosis (DVT). Answer D is second line in patients who are not wanting
    contraceptive therapy; however, this patent is contraindicated
    because of her DVT history
40
Q
  1. A 38-year-old woman and her partner have been unsuccessful in achieving a pregnancy after 10 months including
    three cycles of letrozole (including this month). Her menses started 3 days ago. Her medical history is significant
    for WHO class 2 infertility, PCOS, asthma, and anxiety. Initial evaluation of fertility revealed no additional significant
    findings in either partner. Her current medications include a prenatal vitamin, albuterol 2 puffs every 6 hours
    as needed for asthma, fluticasone 44 mcg 2 puffs twice daily, and buspirone 15 mg twice daily. Her laboratory test
    results were within normal limits. She has an allergy to latex. Which is the best initial therapy for her infertility?
    A. Add metformin.
    B. Change to follitropin-alfa.
    C. Change to clomiphene.
    D. Add progesterone for 10 days.
A
  1. Answer: B
    Because this patient has WHO class 2 infertility, letrozole
    would be the best initial treatment. However, because
    the patient’s letrozole therapy has failed with an adequate
    3-month trial, the next step would be to add gonadotropin
    therapy (Answer B is correct). This will provide FSH.
    Answer C is incorrect because letrozole has higher efficacy
    than clomiphene in this population. Answer A is incorrect
    because metformin can be added to clomiphene as an
    alternative therapy but is not recommended with letrozole.
    Answer D is incorrect because the patient’s menses started
    3 days ago. Short courses of progesterone can be used to
    induce menses for patients; however, newer evidence has
    shown that providers can start another cycle without inducing
    bleeding if it is certain the patient is pregnant.
41
Q
  1. A patient is 18 weeks pregnant and here for her prenatal visit. You discover that she has not been taking lamotrigine
    because of a fear of birth defects. Her last seizure was 6 months ago. Which is best for educating the patient
    on the risk of birth defects?
    A. Risk is low because she is past the stage when cleft palate/lip develops.
    B. Risk is low because intrauterine growth restriction is similar to uncontrolled epilepsy.
    C. Risk is high because lamotrigine is associated with the development of cardiac abnormalities during second
    trimester.
    D. Risk is high because neurodevelopmental delays are associated with exposure to lamotrigine during the
    second and third trimesters.
A
  1. Answer: A
    Lamotrigine is associated with an increased risk of cleft
    palate/lip; however, the palate and lip are developed by
    9 weeks’ gestation. The patient is currently at 18 weeks’
    gestation, so lamotrigine could not cause cleft palate/lip
    (Answer A). Lamotrigine has not been associated with
    intrauterine growth restriction, cardiac abnormalities, or
    neurodevelopment delays (Answers B, C, and D).
42
Q
  1. A 44-year-old woman is experiencing vasomotor symptoms that disrupt her ability to complete work activities
    and get a good night’s sleep. Her medical history is significant for hyperlipidemia and total abdominal hysterectomy
    with bilateral salpingo-oophorectomy (2 months ago). Laboratory values include total cholesterol 198 mg/
    dL, triglycerides 225 mg/dL, and high-density lipoprotein cholesterol 44 mg/dL. Which product is most appropriate
    for this patient?
    A. Estinyl tablet (ethinyl estradiol) 0.02 mg/day.
    B. Vivelle patch (17β-estradiol) 0.025 mg/day twice weekly.
    C. Prempro tablet (conjugated equine estrogens plus medroxyprogesterone) 0.45 mg/1.5 mg/day.
    D. Estratest tablet (esterified estrogens plus testosterone) 0.625 mg/1.25 mg/day.
A
  1. Answer: B
    The patient does not require progestin therapy because
    she has had her uterus removed (Answer C). Testosterone
    therapy is not required because she is not experiencing
    decreased libido with a diagnosis of hypoactive sexual
    dysfunction disorder (Answer D). A non-oral route would
    be preferred because of her elevated triglyceride values
    (Answer B). Oral therapy has the highest effects on the
    cholesterol panel, including increasing the concentration of
    triglycerides (Answer A).
43
Q
  1. A 66-year-old woman is experiencing vaginal dryness and painful intercourse. She states her symptoms are
    bothersome but not horrible. She would like therapy to help relieve the symptoms. Her medical history includes
    hyperlipidemia, hypertension, low bone mass, and coronary artery disease (myocardial infarction 3 years ago).
    Her allergies include adhesives (reaction is rash). Which is the best recommendation?
    A. Estrogel (17β-estradiol gel) 0.75 mg daily.
    B. Venlafaxine XR (extended release) 37.5 mg daily.
    C. Ospemifene 60 mg daily.
    D. Lubricating gel three times daily.
A
  1. Answer: D
    The options in Answers A and C are contraindicated for
    this patient because of her history of myocardial infarction.
    In addition, Answer A is incorrect because this patient
    still has her uterus and would need a progestogen with the
    dyspareunia, it would be inappropriate for this patient
    (Answer C). Answer B is incorrect because this is a nonhormonal
    product used for vasomotor, not genitourinary,
    symptoms. Patients with mild to moderate genitourinary
    symptoms should start with a vaginal lubricating gel
    (Answer D).estrogen for protection against endometrial hyperplasia.
    Although ospemifene is indicated for moderate to severe